Math, asked by pradnyashahade14, 8 months ago

plz slove
urgent hai
I will mark you
as brainliest.


Attachments:

Answers

Answered by Anonymous
121

\large\pink{\underline{\underline{\mathfrak{\green{\sf{QUESTION}}}}}}.

Prove That

\mapsto\pink{\sf{\dfrac{(1+\sec \theta - \tan \theta)}{(1+\sec \theta + \tan \theta)}\:=\:\left(\dfrac{1-\sin \theta}{\cos \theta}\right)}}

\large\pink{\underline{\underline{\mathfrak{\green{\sf{PROVE}}}}}}.

L.H.S.

\mapsto\pink{\sf{\dfrac{(1+\sec \theta - \tan \theta)}{(1+\sec \theta + \tan \theta)}}}

\:\:\:\:\:\green{\sf{\:(\sec^2 \theta - \tan^2 \theta\:=\:1)}}

\mapsto\sf{\dfrac{(\sec^2 \theta - \tan^2 \theta)+(\sec \theta - \tan \theta}{(1+\sec \theta + \tan \theta)}}

\:\:\:\:\:\green{\sf{\:(a^2-b^2)\:=\:(a+b)(a-b)}}

\mapsto\sf{\dfrac{(\sec \theta - \tan \theta)(\sec \theta + \tan \theta)+(\sec \theta - \tan \theta)}{(1+\sec \theta + \tan \theta)}}

\:\:\:\:\:\green{\sf{\:(take\:common)}}

\mapsto\sf{\dfrac{(\sec \theta - \tan \theta)\cancel{(\sec \theta + \tan \theta + 1)}}{\cancel{(1+\sec \theta + \tan \theta )}}}

\mapsto\sf{\:(\sec \theta - \tan \theta)}

\:\:\:\:\:\green{\sf{\:\left(\sec \theta \:=\dfrac{1}{\cos \theta}\:\:and\:\tan \theta \:=\dfrac{\sin \theta}{\cos \theta}\right)}}

\mapsto\sf{\dfrac{1}{\cos \theta}\:-\dfrac{\sin \theta}{\cos \theta}}

\mapsto\pink{\sf{\left(\dfrac{1-\sin \theta}{\cos \theta}\right)}}

\:\:\:\:\red{\sf{\:=RHS}}

Proved

________________,_,____

Answered by sandy1816
4

Answer:

your answer attached in the photo

Attachments:
Similar questions